Test 11: Critical Reading Answers
Section 2
2. ANSWERS AND EXPLANATIONS
1. ANSWERS AND EXPLANATIONS
Choice (C) is correct. "Muddled" means confused or
aimless. "Clarity" means clearness. If one were to insert
these terms into the text, the sentence would read
"Responding to criticism that the script was rambling
and muddled, the new screenwriter revised the
dialogue for greater succinctness and clarity." The
missing terms describe a second negative quality of the
script and a corresponding positive quality of the
revised version. A muddled script would need to be
revised for clarity.
Choice (A) is incorrect. "Engaging" means interesting.
"Simplicity" means uncomplicated. If one were to insert
these terms into the text, the sentence would read
"Responding to criticism that the script was rambling
and engaging, the new screenwriter revised the
dialogue for greater succinctness and simplicity." The
missing terms describe a second negative quality of the
script and a corresponding positive quality of the
revised version. A screenplay would not be criticized for
being engaging.
Choice (B) is incorrect. "Subjective" means dependent
on personal taste or views. "Ambiguity" means
uncertainty. If one were to insert these terms into the
text, the sentence would read "Responding to criticism
that the script was rambling and subjective, the new
screenwriter revised the dialogue for greater
succinctness and ambiguity." The missing terms
describe a second negative quality of the script and a
corresponding positive quality of the revised version. If
a screenplay were criticized for being subjective, the
screenwriter would probably revise it to make it less
ambiguous.
Choice (D) is incorrect. "Terse" means brief or succinct.
"Emptiness" means, in this context, meaninglessness.
If one were to insert these terms into the text, the
sentence would read "Responding to criticism that the
script was rambling and terse, the new screenwriter
revised the dialogue for greater succinctness and
emptiness." The missing terms describe a second
negative quality of the script and a corresponding
positive quality of the revised version. A screenplay
cannot be both rambling and terse, and one would not
revise it to give it greater emptiness.
Choice (E) is incorrect. "Difficult" means hard.
"Abstraction" means theoretical. If one were to insert
these terms into the text, the sentence would read
"Responding to criticism that the script was rambling
and difficult, the new screenwriter revised the dialogue
for greater succinctness and abstraction." The missing
terms describe a second negative quality of the script
and a corresponding positive quality of the revised
version. If a screenplay is difficult, then adding more
abstraction would only make it more difficult.
Choice (E) is correct. "Renaissance" means a rebirth or
revival. If one were to insert this term into the text, the
sentence would read "During the 1990ʼs, Shanghai
benefited from an architectural renaissance, the result
of a dramatic increase in innovative and artistic
building." An "architectural renaissance" describes a
period of renewed excitement in the field of
architecture, and properly describes the result of a
dramatic increase in new artistic construction.
Choice (A) is incorrect. "Intransigence" means being
uncompromising or stubborn. If one were to insert this
term into the text, the sentence would read "During the
1990ʼs, Shanghai benefited from an architectural
intransigence, the result of a dramatic increase in
innovative and artistic building." It is unlikely that
"intransigence" would be the result of an increase in
architectural innovation; innovation is in some ways the
opposite of stubbornness.
Choice (B) is incorrect. A "plenitude" means an
abundance. If one were to insert this term into the text,
the sentence would read "During the 1990ʼs, Shanghai
benefited from an architectural plenitude, the result of a
dramatic increase in innovative and artistic building." An
abundance of architecture would be the result of an
increase in construction. But this would not account for
the "innovative and artistic" quality of the architecture.
Choice (C) is incorrect. A "desecration" means an
outrageous mistreatment of something—the
vandalizing of a sacred building, for instance. If one
were to insert this term into the text, the sentence
would read "During the 1990ʼs, Shanghai benefited
from an architectural desecration, the result of a
dramatic increase in innovative and artistic building."
Desecration would not benefit a city.
Choice (D) is incorrect. "Stagnation" means a lack of
movement or development. If one were to insert this
term into the text, the sentence would read "During the
1990ʼs, Shanghai benefited from an architectural
stagnation, the result of a dramatic increase in
innovative and artistic building." Stagnation is the exact
opposite of a dramatic increase in innovation.
3. ANSWERS AND EXPLANATIONS
Choice (B) is correct. "Elusive" means hard to find.
"Imperceptible" means difficult to perceive or detect. If
one were to insert these terms into the text, the
sentence would read "Many subatomic nuclear
particles are elusive and nearly imperceptible: they are
hard to track as well as to detect." The statement
following the colon explains or expands on the
statement preceding it. The first missing term indicates
that the particles are hard to track: this perfectly defines
"elusive." The second missing term indicates that they
Page 1 of 21
Test 11: Critical Reading Answers
are hard to detect: this perfectly defines
"imperceptible."
Choice (A) is incorrect. "Unstable" means likely to
change suddenly. "Explosive" means likely to blow up.
If one were to insert these terms into the text, the
sentence would read "Many subatomic nuclear
particles are unstable and nearly explosive: they are
hard to track as well as to detect." The statement
following the colon explains or expands on the
statement preceding it. The first missing term indicates
that the particles are hard to track: this does not define
"unstable."
Choice (C) is incorrect. "Minute" means extremely
small. "Immobile" means not moving. If one were to
insert these terms into the text, the sentence would
read "Many subatomic nuclear particles are minute and
nearly immobile: they are hard to track as well as to
detect." The statement following the colon explains or
expands on the statement preceding it. The first
missing term indicates that the particles are hard to
track: this does not precisely define minute. The second
missing term indicates that they are hard to detect: this
does not define immobile. Anything that does not move
should be easy, not difficult, to detect.
Choice (D) is incorrect. "Charged" means full of energy.
"Reactive" means readily responsive to a stimulus. If
one were to insert these terms into the text, the
sentence would read "Many subatomic nuclear
particles are charged and nearly reactive: they are hard
to track as well as to detect." The statement following
the colon explains or expands on the statement
preceding it. The first missing term indicates that the
particles are hard to track: this does not define charged.
The second missing term indicates that they are hard to
detect: this does not define reactive.
Choice (B) is incorrect. "Intrusive" means invasive or
nosy. If one were to insert this term into the text, the
sentence would read "The crafty child tricked his
innocent brother, a particularly intrusive and trusting
boy, into committing a mischievous prank." The missing
term characterizes a brother who is innocent, trusting,
and able to be tricked. A brother who is intrusive may or
may not be such a person.
Choice (C) is incorrect. "Astute" means knowledgeable
and wise. If one were to insert this term into the text,
the sentence would read "The crafty child tricked his
innocent brother, a particularly astute and trusting boy,
into committing a mischievous prank." The missing term
characterizes a brother who is innocent, trusting, and
able to be tricked. An astute person would not likely be
tricked.
Choice (D) is incorrect. "Opportunistic" means taking
advantage of opportunities when they arise. If one were
to insert this term into the text, the sentence would read
"The crafty child tricked his innocent brother, a
particularly opportunistic and trusting boy, into
committing a mischievous prank." The missing term
characterizes a brother who is innocent, trusting, and
able to be tricked. Someone who is opportunistic may
or may not be such a person.
Choice (E) is incorrect. "Circumspect" means
possessing caution or prudence. If one were to insert
this term into the text, the sentence would read "The
crafty child tricked his innocent brother, a particularly
circumspect and trusting boy, into committing a
mischievous prank." The missing term characterizes a
brother who is innocent, trusting, and able to be tricked.
It would be hard to trick a circumspect person.
5. ANSWERS AND EXPLANATIONS
Choice (E) is incorrect. "Tenuous" means flimsy.
"Indivisible" means not capable of being broken down
into smaller parts. If one were to insert these terms into
the text, the sentence would read "Many subatomic
nuclear particles are tenuous and nearly indivisible:
they are hard to track as well as to detect." The
statement following the colon explains or expands on
the statement preceding it. The first missing term
indicates that the particles are hard to track: this does
not define "tenuous." The second missing term
indicates that they are hard to detect: this does not
define "indivisible."
4. ANSWERS AND EXPLANATIONS
Choice (A) is correct. "Guileless" means innocent and
naive. If one were to insert this term into the text, the
sentence would read "The crafty child tricked his
innocent brother, a particularly guileless and trusting
boy, into committing a mischievous prank." The missing
term characterizes a brother who is innocent, trusting,
and able to be tricked. "Guileless" suggests such a
person.
Choice (E) is correct. "Deftness" means skill or
dexterity. If one were to insert this term into the text, the
sentence would read "Ellen Ochoa's deftness with the
apparatus in the space shuttle Discovery was apparent
when she adroitly manipulated the shuttle's robot arm."
The missing term must have a meaning close to that of
"adroit," which means skillful or dexterous. Deft and
adroit are synonymous.
Choice (A) is incorrect. "Compromise" is a settlement of
mutual concessions. If one were to insert this term into
the text, the sentence would read "Ellen Ochoa's
compromise with the apparatus in the space shuttle
Discovery was apparent when she adroitly manipulated
the shuttle's robot arm." The missing term must have a
meaning close to that of "adroit," which means skillful or
dexterous. A compromise in this context has nothing to
do with an adroit action.
Choice (B) is incorrect. "Humility" is a modest attitude.
If one were to insert this term into the text, the sentence
would read "Ellen Ochoa's humility with the apparatus
Page 2 of 21
Test 11: Critical Reading Answers
in the space shuttle Discovery was apparent when she
adroitly manipulated the shuttle's robot arm." The
missing term must have a meaning close to that of
"adroit," which means skillful or dexterous. Humility has
nothing to do with performing an adroit action.
Choice (C) is incorrect. "Machinations" are crafty
schemes. If one were to insert this term into the text,
the sentence would read "Ellen Ochoa's machinations
with the apparatus in the space shuttle Discovery was
apparent when she adroitly manipulated the shuttle's
robot arm." The missing term must have a meaning
close to that of "adroit," which means skillful or
dexterous. Machinations have nothing to do with an
adroit action.
Choice (D) is incorrect. "Synergy" describes an action
or operation that is greater than the sum of its parts. If
one were to insert this term into the text, the sentence
would read "Ellen Ochoa's synergy with the apparatus
in the space shuttle Discovery was apparent when she
adroitly manipulated the shuttle's robot arm." The
missing term must have a meaning close to that of
"adroit," which means skillful or dexterous. Synergy has
nothing to do with an adroit action.
6. ANSWERS AND EXPLANATIONS
Choice (B) is correct. Each passage specifically
addresses readers' abilities to understand writing.
Passage 1 addresses journalistic practices of writing
short, simple sentences designed to be understood by
a large audience. Passage 2 challenges the idea that
readers can only handle short, simple sentences.
Choice (A) is incorrect. Although both passages are
about people who write, neither addresses why people
choose to write as a career.
Choice (B) is incorrect. The statement in Passage 2
that most readers "have no patience" with complex
writing supports Passage 1's description of effective
prose; in fact, Passage 1 uses this same argument.
Choice (C) is incorrect. Both passages agree that, as
Passage 2 concedes, "clarity and succinctness," or
conciseness, are "worthy goals."
Choice (D) is incorrect. The statement in Passage 2
that people have "no interest" in "challenging prose"
supports the argument for simple prose presented in
Passage 1.
8. ANSWERS AND EXPLANATIONS
Choice (A) is correct. The author of Passage 2 argues
that readers would like challenging writing if only it were
given to them. Consequently, the author of Passage 2
would likely show concern, or unease, with Passage 1's
scornful description of readers. The author states that "I
sometimes worry" that a negative view of readers is
being taught to young writers.
Choice (B) is incorrect. The author of Passage 2 would
not likely be perplexed, or confused, by the views
expressed at the end of Passage 1, but would simply
disagree with them.
Choice (C) is incorrect. Disdain, or scorn, would be
inconsistent with the reasonable and concerned tone of
Passage 2.
Choice (D) is incorrect. Passage 2 does not use humor
in framing an argument that challenges the commonly
held negative view of readers; rather, the author is
concerned about the negative view of readers that is
affecting the way young writers compose their work.
Choice (C) is incorrect. Although admitting readers
may prefer a certain style of writing, neither passage
says anything about how writers know what readers
prefer or that readers convey, or communicate, their
preferences to writers.
Choice (E) is incorrect. The author of Passage 2 would
likely disagree with the opinions in Passage 1, rather
than showing appreciation.
Choice (D) is incorrect. Neither passage discusses
different levels of writing experience.
Choice (E) is correct. Passage 1 mocks the "experts"
and also its own subject (writing) by using long
sentences and uncommon words to explain that good
writing should do the opposite. The language is
especially sarcastic when the author portrays the
"experts'" treatment of writing as a science that has
"perennial and fundamental laws" that can be
calculated "apparently with the mathematical precision
of astronomers." The tone of Passage 2 is concerned,
not sarcastic. Passage 2 speaks of the responsibility of
writers to raise the reading level of their audience.
Choice (E) is incorrect. Both passages acknowledge
the common belief that short sentences are easier to
understand than long ones.
7. ANSWERS AND EXPLANATIONS
Choice (E) is correct. By stating that it is writers'
responsibility to offer "challenging prose," Passage 2
contradicts the assertion in Passage 1 that "effective
prose" must be very simple.
Choice (A) is incorrect. Both statements say the same
thing, that effective prose is simple and straightforward.
9. ANSWERS AND EXPLANATIONS
Choice (A) is incorrect. The tone of Passage 1 is
sarcastic and cutting when it describes the certainty of
"experts" on the essence of good writing, which is the
opposite of being earnest, or sincere. Passage 2 clearly
Page 3 of 21
Test 11: Critical Reading Answers
expresses earnest, or heartfelt, feelings about the
practice of writing and the capabilities of readers.
that animals are concerned with their physical needs
before all others.
Choice (B) is incorrect. Passage 1 presents negative
opinions about readers, which could hardly be called
inspirational. Passage 2, on the other hand, does
attempt to inspire young writers to believe that readers
are better than they think and that they can read more
complex prose.
Choice (B) is incorrect. Although ensuring that one has
enough food could be called a "survival skill," the use of
the motto, "Grub first, then ethics," in this passage does
not suggest that these skills have been forced on
animals. Furthermore, the passage never discusses
survival skills.
Choice (C) is incorrect. The tone of ridicule in Passage
1 is far from complacent, or easy-going. Neither of the
passages are complacent, or untroubled; Passage 1 is
sarcastic, or cutting, and Passage 2 is more earnest
and inspirational.
Choice (C) is incorrect. Although animals may desire a
consistent routine, the motto is about food, not
consistency.
Choice (D) is incorrect. Neither passage is defensive in
tone; Passage 1 is mocking in tone, while Passage 2
makes a reasoned argument.
10. ANSWERS AND EXPLANATIONS
Choice (C) is correct. The examples given in the lines
"the golden retriever rolling the grass, the horse with his
nose deep in the oats, and kitty by the fire" are all
presented as examples of "creature comforts," images
that evoke, or bring to mind, notions of happiness or
contentment.
Choice (A) is incorrect. Although the examples in lines
7-8 do show animals engaged in a variety of activities,
this is not their primary purpose. Their primary purpose
is to offer examples of animal activities that humans
believe indicate that animals are content.
Choice (B) is incorrect. Although the passage argues
that animal happiness differs from human happiness, it
does not suggest that the examples presented in lines
7-8 are deceptive, or misleading.
Choice (D) is incorrect. The passage does not suggest
that anyone would be likely to find its arguments
"implausible," or hard to believe.
Choice (E) is incorrect. The author does not mention
nostalgic longings, even though the activities described
could certainly evoke those feelings in readers.
11. ANSWERS AND EXPLANATIONS
Choice (E) is correct. In the context of the passage, the
motto, "Grub first, then ethics," means that it is
important to satisfy physical needs before one
addresses ethical or moral dilemmas. For animals, this
means that they tend to be concerned about physical
needs first.
Choice (A) is incorrect. Although the passage says that
the motto, "Grub first, then ethics," "would describe
many a wise Labrador retriever" (lines 10-11), there is
no suggestion that such an animal would be any wiser
than would be expected. The motto simply suggests
Choice (D) is incorrect. The passage does not address
the topic of animals' relationships with people.
12. ANSWERS AND EXPLANATIONS
Choice (B) is correct. "Temperament" means
disposition or character. The author feels that the
careers "of a good woodcarver or a dancer or a poet"
are more consistent with his own temperament than a
career in business. This suggests that the author
believes artistic careers to be the most "personally
fulfilling of all endeavors."
Choice (A) is incorrect. Although the author does
mention a poet as one of the examples in lines 17-21,
the issue of whether or not a poet could have a
successful business career is not addressed.
Choice (C) is incorrect. In lines 17-21, the author does
not discuss the issue of whether or not "a busy life can
have its own rewards." Instead, the author seems
concerned with the satisfaction that can be gained
through work that is personally fulfilling (such as the
work done by a "good woodcarver or a dancer or a
poet").
Choice (D) is incorrect. Although the author may hold
the view that "few people are ever satisfied with the
jobs they have chosen," this view is not addressed in
the passage. On the contrary, the passage suggests
that work can be an extremely rewarding pursuit.
Choice (E) is incorrect. The author probably believes
this to be true, but it is not relevant to the author's
discussion of temperament in lines 17-21.
13. ANSWERS AND EXPLANATIONS
Choice (A) is correct. The statement points out that a
social circumstance, namely musical education,
enabled Mozart's genius to find expression. The
passage also suggests that without this musical
education Mozart's natural talent may not have found
expression.
Choice (B) is incorrect. Although teaching young
children to play the harpsichord might be described as
pushing them to excel, the primary focus of the
Page 4 of 21
Test 11: Critical Reading Answers
discussion in lines 25-28 is on the role of musical
education in the development of musical genius.
Choice (A) is incorrect. Although finding a mate could
be thought of as contributing to happiness, the kind of
happiness the author is discussing involves work.
Choice (C) is incorrect. The author makes no comment
about the relative incidence, or numbers of
occurrences, of genius in the past or in the present.
Choice (D) is incorrect. Even though the harpsichord
may have been the ideal musical instrument for
Mozartʼs early talent, the emphasis in the passage is
not on which instrument he played, but on his having
been given a musical education.
Choice (E) is incorrect. Although the author discusses
the pleasure that artists derive from achievement earlier
in the passage, in this section the emphasis is on the
role of musical education in the development of
Mozart's talent.
Choice (C) is incorrect. "A horse being carefully
groomed for a show" is a creature receiving attention,
not one that is working or performing a challenging
task.
Choice (D) is incorrect. Although escaping from a zoo
might be pleasurable as well as challenging for a
monkey, it is not the kind of "work in the full sense" (line
15) that would give the animal pleasure for a job well
done.
Choice (E) is incorrect. Caring for kittens is an action of
devotion, but it is not work or a career in the sense
discussed in the passage.
14. ANSWERS AND EXPLANATIONS
16. ANSWERS AND EXPLANATIONS
Choice (C) is correct. In the passage, the author
describes "animal versions, if not equivalents, of
Mozart," arguing that "they cannot make their
spontaneous passions into sustained happiness without
education, any more than Mozart could have." The
author uses the word "passions" in the sense of
impulses, talents, or enthusiasms that require training
and education if they are to lead to "happiness."
Choice (C) is correct. The author writes, "[w]hat did
amaze me about the potato-size rock that fell from
Mars was that it had traveled millions of miles to land
here" (lines 5-7). Further, the author describes the
rock's journey as one in which it was "blasted from
world to world," and had "lain waiting for milennia upon
an Antarctic ice field" (lines 7-10). The author is clearly
amazed at the "seemingly unlikely sequence of events"
that led to the rock's discovery.
Choice (A) is incorrect. "Passions" does not refer to
love or ardent affections in this context, but to strong
impulses, or enthusiasms, such as Mozart's feeling for
music.
Choice (B) is incorrect. In this context, "passions" refer
to a talent, or enthusiasm, that could be cultivated into
happiness through education. Consequently, it would
be strange to say that animals "cannot make" their
violent outbursts "into sustained happiness without
education."
Choice (A) is incorrect. The passage does not mention
whether the scientists had "doubts that such an object
could reach earth." The author expresses her awe or
wonder over the process by which the meteorite came
to earth and was found, but this has little to do with the
beliefs of scientists.
Choice (B) is incorrect. The passage does not mention
the expedition party's expectations or attitudes prior to
finding the rock.
Choice (D) is incorrect. "Passions" does not refer to
prejudices or strong opinions in this context; rather, it
refers to positive impulses, such as talents, that have
the possibility of being shaped into masterpieces like
Mozart's music.
Choice (D) is incorrect. Far from contradicting a "longstanding scientific theory," the "apparent fossils of an
extinct alien form of life" (lines 2-3) in the rock tend to
confirm the author's belief that life may exist on other
planets.
Choice (E) is incorrect. The context does not mention
suffering nor does the passage have anything to do
with suffering.
Choice (E) is incorrect. It is not surprising that such a
rare find would be studied so closely.
17. ANSWERS AND EXPLANATIONS
15. ANSWERS AND EXPLANATIONS
Choice (B) is correct. The passage states, "there is
something more" to animals' happiness than mere
comfort, namely, the "satisfactions that come from
work." A dog herding sheep into a pen is performing the
kind of work that the author describes as satisfying to
animals.
Choice (B) is correct. "Designated" means named. The
passage makes it clear that ALH 84001 is the name
given to the rock by scientists.
Choice (A) is incorrect. The term ALH 84001 is not a
picture, drawing, or means of depicting an object. ALH
84001 is a name.
Page 5 of 21
Test 11: Critical Reading Answers
Choice (C) is incorrect. "Stipulated" means required or
demanded. There is no mention that the naming of the
rock ALH 84001 was stipulated, required, or demanded.
Choice (D) is incorrect. "Selected" means chosen, and
its use in this context would imply that there were other
rocks to choose from. The author makes the unique
nature of this rock clear.
Choice (E) is incorrect. "Allocated" means gave or
assigned. Line 15 does not refer to any allocation or
assignment; rather, it refers to the name given to the
rock.
Choice (E) is incorrect. The author uses the phrase
"this new intimacy" to refer to increasing knowledge
about our solar system. The author does not discuss
whether events happening on one planet influence
events on another.
20. ANSWERS AND EXPLANATIONS
Choice (B) is correct. "Crude" means rough or inexpert.
The author's diorama and its materials—"marbles, jack
balls, and Ping-Pong balls" (lines 30-31)—could well be
described as "crude," or rough and inexpert, especially
since the author used everyday materials in an attempt
to model the solar system.
18. ANSWERS AND EXPLANATIONS
Choice (C) is correct. The author views the scientists'
ability to use minute details of chemical composition to
match the rock found in Antarctica to rocks from
particular regions studied on Mars as "bold," or
confident.
Choice (A) is incorrect. The passage does not offer
conflicting answers to the question of the rock's origin;
rather, the author explains that researchers "think they
have pinpointed its [the rock's] former resting place to
just two possible sites."
Choice (B) is incorrect. The author does not mention a
"young geologist" in the passage.
Choice (D) is incorrect. Researchers were concerned
with the rock's place of origin (lines 20-23), not its
Antarctic discovery site.
Choice (E) is incorrect. The author does not mention
comets or theories about them.
19. ANSWERS AND EXPLANATIONS
Choice (D) is correct. In line 28, "intimacy" means close
acquaintance. The author is referring to how much
scientists have learned about the solar system in recent
decades as compared to what was known 40 years ago
(lines 33-35).
Choice (A) is incorrect. The context of the phrase "this
new intimacy" refers to how much scientists have
learned about our solar system, not their "learning
experience."
Choice (B) is incorrect. Although the author does use
herself and her daughter (both nonspecialists) as
examples in this passage, her use of the phrase "this
new intimacy" refers to what astronomers (specialists)
know about our solar system.
Choice (C) is incorrect. In the context of the passage,
the phrase "this new intimacy" specifically refers to the
solar system and does not allude to the possibility of life
on Mars.
Choice (A) is incorrect. The materials mentioned
("marbles, jack balls, and Ping-Pong balls") were
manufactured items, not natural ones.
Choice (C) is incorrect. "Obvious" means easily
perceived or understood. The author's description
emphasizes the fact that the materials used in the
diorama were rough and unsophisticated, not the idea
that they were easily perceived or understood.
Choice (D) is incorrect. "Vulgar" means indecent or in
poor taste. The passage does not suggest that the
materials used in the diorama were either indecent or in
poor taste.
Choice (E) is incorrect. On the contrary, the author's
account suggests that the materials used in the
diorama were chosen quite specifically to represent
objects in the solar system.
21. ANSWERS AND EXPLANATIONS
Choice (A) is correct. The author refers to Pluto's moon
in order to compare our current knowledge of our solar
system with that of the past. The author's daughter
would need to include Pluto's moon if she were to build
a diorama, because the moon's existence is now
current knowledge.
Choice (B) is incorrect. Pluto's moon was absent from
the author's diorama not because it was too small, but
because it had not yet been discovered.
Choice (C) is incorrect. The author is comparing past
and present knowledge about objects in space, not the
objects themselves.
Choice (D) is incorrect. The author is not comparing the
children's curiosity. Instead, the author is comparing the
knowledge of today's children with that of children
years ago.
Choice (E) is incorrect. Although the author might well
be in favor of continued space exploration, the idea is
not mentioned in the passage.
22. ANSWERS AND EXPLANATIONS
Page 6 of 21
Test 11: Critical Reading Answers
Choice (B) is correct. "Extrapolate" means infer. The
passage states, "we know about them [the planets]
indirectly through the gravitational effects they exert on
their parent stars" (51-53). In other words, the passage
describes planets that are known only by their
gravitational effects. This knowledge would have to be
arrived at by extrapolation, or inference, from
observations of known planets and the gravitational
effects of their planet stars.
Choice (A) is incorrect. The author mentions that the
basis for knowledge about these planets is the
"gravitational effects" these planets "exert on their
parent stars" (lines 52-53). Thus the basis of knowledge
about these planets is the theory of gravity. This is not
an untested theory.
Choice (C) is incorrect. The author does not mention
the number of observers involved. Further, it seems
unlikely that the number of observers involved would
alter any description of the reasoning process.
Choice (D) is incorrect. The passage states that none
of the planets have "actually been seen through a
telescope" (lines 50-51). Thus none of the planets
mentioned in lines 49-53 have been directly observed.
Choice (E) is incorrect. Although the author mentions
the discovery of a rock from Mars (lines 5-6), no
physical evidence has been collected to support the
existence of the planets described in lines 49-53.
23. ANSWERS AND EXPLANATIONS
Choice (A) is correct. The planet was nicknamed
"Goldilocks" because its temperature was "just right," or
perfect, for the presence of liquid water. The possibility
of liquid water, "not known to exist anywhere in our
solar system now except on Earth, is thought crucial to
biological life." Scientists are eager to find a planet with
liquid water because it could support life.
Choice (B) is incorrect. The passage does not say what
scientists previously thought about the atmospheric
temperature of the planet.
Choice (C) is incorrect. In lines 53-59, the author uses
the fairy tale to make a point about what Goldilocks
found (things that were "just right"), not to make a point
about the methods that Goldilocks used.
Choice (D) is incorrect. Though the planet has not been
observed visually, the author does not express any
doubt that it exists. Further, the author describes
scientists' means of discovering this and other planets:
"we know about them indirectly through the
gravitational effects they exert on their parent
stars" (lines 51-53).
Choice (E) is incorrect. Although Goldilocks did find
things that were "just right" through trial and error, the
author does not mention the use of trial and error in her
discussion of this planet's location. Instead, the author
simply states that the planet was found by measuring
the gravitational effects exerted on its parent star (lines
51-53).
24. ANSWERS AND EXPLANATIONS
Choice (C) is correct. Since water "is thought crucial to
biological life" (line 61), evidence that Mars was once
wet would be seen as suggesting that life could have
existed there.
Choice (A) is incorrect. Although the passage does
mention the possibility that Mars may have experienced
a "planetary collision of the sort that purportedly killed
off our dinosaurs" (lines 8-9), the passage does not
suggest that such a planetary collision would have any
bearing on Mars's capacity to support life.
Choice (B) is incorrect. A mild atmospheric temperature
need not necessarily include water, which is essential
to life. Furthermore, there is no mention of the effects of
a mild atmospheric temperature on the development of
life on Mars.
Choice (D) is incorrect. The passage does not say
whether rocks in Antarctica have any properties that
would suggest the potential for life, or that rocks in the
Antarctic ice field bear any resemblance, or likeness, to
rocks from Mars.
Choice (E) is incorrect. Fossils in a rock, which the
author describes as "suggestive traces of something
that might once have lived and died" (lines 26-27), are
mentioned as evidence suggestive of life. Fissures, or
cracks, are not.
-------------------------------------------------------
Section 5
1. ANSWERS AND EXPLANATIONS
Choice (E) is correct. "Conjunction with" means
together with. "Trace" means track or follow. If one were
to insert these terms into the text, the sentence would
read "Much of our knowledge of dinosaurs comes from
excavated bones, which, in conjunction with other clues
such as fossilized tracks and eggs, help us to trace the
evolution of these creatures." The first missing term
suggests that knowledge and clues come together. The
second missing term indicates what the knowledge and
clues enable people to do. It makes sense to say that
knowledge and clues work together to enable people to
trace the evolution of dinosaurs.
Choice (A) is incorrect. "Convergence with" means
come together with. "Supplant" means replace. If one
were to insert these terms into the text, the sentence
would read "Much of our knowledge of dinosaurs
Page 7 of 21
Test 11: Critical Reading Answers
comes from excavated bones, which, in convergence
with other clues such as fossilized tracks and eggs,
help us to supplant the evolution of these creatures."
The first missing term suggests that knowledge and
clues come together. The second missing term
indicates what the knowledge and clues enable people
to do. Supplant does not make sense in the sentence
because evolution cannot be replaced.
Choice (B) is incorrect. "Divergence from" means
departing from. "Decode" means to put a coded
message into plain language. If one were to insert
these terms into the text, the sentence would read
"Much of our knowledge of dinosaurs comes from
excavated bones, which, in divergence from other clues
such as fossilized tracks and eggs, help us to decode
the evolution of these creatures." The first missing term
suggests that knowledge and clues come together. The
second missing term indicates what the knowledge and
clues enable people to do. Knowledge and clues would
enable people to decode how dinosaurs evolved
However, "divergence from" does not suggest that the
knowledge and clues are coming together.
Choice (C) is incorrect. "Dependence on" means
leaning on. "Belie" means contradict or fail to confirm. If
one were to insert these terms into the text, the
sentence would read "Much of our knowledge of
dinosaurs comes from excavated bones, which, in
dependence on other clues such as fossilized tracks
and eggs, help us to belie the evolution of these
creatures." The first missing term suggests that
knowledge and clues come together. The second
missing term indicates what the knowledge and clues
enable people to do. Bones themselves would not
depend on clues. Additionally, while knowledge and
clues might belie, or contradict, evolutionary theories,
they do not belie evolution itself.
Choice (D) is incorrect. "Opposition to" means against.
"Amplify" means make larger or greater. If one were to
insert these terms into the text, the sentence would
read "Much of our knowledge of dinosaurs comes from
excavated bones, which, in opposition to other clues
such as fossilized tracks and eggs, help us to amplify
the evolution of these creatures." The first missing term
suggests that knowledge and clues come together. The
second missing term indicates what the knowledge and
clues enable people to do. People cannot amplify the
evolution of creatures.
2. ANSWERS AND EXPLANATIONS
Choice (E) is correct. "Ephemeral" means brief.
"Evaporate" means disappear by turning into vapor. If
one were to insert these terms into the text, the
sentence would read "Vernal pools are among the most
ephemeral of ponds: they form as a result of snowmelt
and a high water table in winter, and then they
evaporate by late summer." "Vernal pools" are spring
pools. The first missing term is a characteristic of a
vernal pool. What follows the colon is an explanation of
the first missing term. Evaporation shows that the
ponds disappear after a period of time. This
corresponds with the use of ephemeral to describe a
vernal pool.
Choice (A) is incorrect. "Transitory" means temporary
or brief. "Expand" means increase in size. If one were
to insert these terms into the text, the sentence would
read "Vernal pools are among the most transitory of
ponds: they form as a result of snowmelt and a high
water table in winter, and then they expand by late
summer." "Vernal pools" are spring pools. The first
missing term is a characteristic of a vernal pool. What
follows the colon is an explanation of the first missing
term. A transitory pond would disappear after a brief
time. However, ponds that expand are not temporary.
Choice (B) is incorrect. "Anachronistic" means from the
wrong time period. "Overflow" means flow over the
limits of something. If one were to insert these terms
into the text, the sentence would read "Vernal pools are
among the most anachronistic of ponds: they form as a
result of snowmelt and a high water table in winter, and
then they overflow by late summer." "Vernal pools" are
spring pools. The first missing term is a characteristic of
a vernal pool. What follows the colon is an explanation
of the first missing term. A pond cannot be
anachronistic.
Choice (C) is incorrect. "Immutable" means
unchangeable. "Drain" means flow away. If one were to
insert these terms into the text, the sentence would
read "Vernal pools are among the most immutable of
ponds: they form as a result of snowmelt and a high
water table in winter, and then they drain by late
summer." "Vernal pools" are spring pools. The first
missing term is a characteristic of a vernal pool. What
follows the colon is an explanation of the first missing
term. A pond that is immutable would not fill in winter
and drain in summer.
Choice (D) is incorrect. "Itinerant" means roaming or
wandering from place to place. "Teem" means become
filled to overflowing. If one were to insert these terms
into the text, the sentence would read "Vernal pools are
among the most itinerant of ponds: they form as a
result of snowmelt and a high water table in winter, and
then they teem by late summer." "Vernal pools" are
spring pools. The first missing term is a characteristic of
a vernal pool. What follows the colon is an explanation
of the first missing term. Itinerant is not described by
the concept of teeming. As well, a pond stays in the
same place; it cannot be itinerant.
3. ANSWERS AND EXPLANATIONS
Choice (B) is correct. "Diverse" means composed of
different elements, and "condensed" means made
compact. If one were to insert these terms into the text,
the sentence would read "The diverse experiences of
Madonna Swan, the 1983 North American Indian
Woman of the Year, cannot be fully appreciated if they
Page 8 of 21
Test 11: Critical Reading Answers
are condensed in a tidy summary." Experiences
combining different elements cannot be easily
compacted in a tidy summary.
Choice (A) is incorrect. "Varied" means mixed, and
"interposed" means inserted. If one were to insert these
terms into the text, the sentence would read "The
varied experiences of Madonna Swan, the 1983 North
American Indian Woman of the Year, cannot be fully
appreciated if they are interposed in a tidy summary."
While varied experiences may, with difficulty, be
summarized, it does not make sense to speak of
experiences being inserted into a summary. That would
suggest that the summary of experiences already
exists.
Choice (C) is incorrect. "Profound" means deep.
"Magnified" means enlarged. If one were to insert these
terms into the text, the sentence would read "The
profound experiences of Madonna Swan, the 1983
North American Indian Woman of the Year, cannot be
fully appreciated if they are magnified in a tidy
summary." While deeply meaningful experiences may
be summarized, a summary is by definition a brief
version of something. It makes no sense to say that
something could be enlarged in a summary.
Choice (D) is incorrect. "Transformative" means
causing a change. "Embellished" means enhanced. If
one were to insert these terms into the text, the
sentence would read "The transformative experiences
of Madonna Swan, the 1983 North American Indian
Woman of the Year, cannot be fully appreciated if they
are embellished in a tidy summary." A summary
presents main points in a general way; it does not
"embellish," or enhance, those points.
Choice (E) is incorrect. "Impressive" means inspiring
awe or admiration, and "immersed" means submerged.
If one were to insert these terms into the text, the
sentence would read "The impressive experiences of
Madonna Swan, the 1983 North American Indian
Woman of the Year, cannot be fully appreciated if they
are immersed in a tidy summary." Although impressive
experiences might be mentioned in a summary, it would
be strange to describe such experiences as being
submerged.
4. ANSWERS AND EXPLANATIONS
Choice (A) is correct. "Orthodox" means conforming to
traditional beliefs. If one were to insert this term into the
text, the sentence would read "The representative was
a traditionalist, reluctant to support any legislation
inconsistent with the nation's most orthodox principles."
The missing term describes the kind of legislative
principles that a "traditionalist," a person who upholds
traditional beliefs, would support. Orthodox principles
are exactly the type of principles a traditionalist would
tend to support.
Choice (B) is incorrect. "Impassioned" means
passionate. If one were to insert this term into the text,
the sentence would read "The representative was a
traditionalist, reluctant to support any legislation
inconsistent with the nation's most impassioned
principles." The missing term describes the kind of
legislative principles that a "traditionalist," a person who
upholds traditional beliefs, would support. It is odd to
say that principles themselves could be impassioned.
This word is usually used to describe people, not things
such as principles. In any case, impassioned principles
would not necessarily be in line with tradition.
Choice (C) is incorrect. "Precarious" means on edge or
unsafe. If one were to insert this term into the text, the
sentence would read "The representative was a
traditionalist, reluctant to support any legislation
inconsistent with the nation's most precarious
principles." The missing term describes the kind of
legislative principles that a "traditionalist," a person who
upholds traditional beliefs, would support. A
traditionalist may or may not support principles that are
precarious.
Choice (D) is incorrect. "Impressionable" means
capable of being easily impressed or swayed. If one
were to insert this term into the text, the sentence
would read "The representative was a traditionalist,
reluctant to support any legislation inconsistent with the
nation's most impressionable principles." The missing
term describes the kind of legislative principles that a
"traditionalist," a person who upholds traditional beliefs,
would support. Impressionable is a word that applies to
people, not to things such as principles.
Choice (E) is incorrect. "Indeterminate" means vague. If
one were to insert this term into the text, the sentence
would read "The representative was a traditionalist,
reluctant to support any legislation inconsistent with the
nation's most indeterminate principles." The missing
term describes the kind of legislative principles that a
"traditionalist," a person who upholds traditional beliefs,
would support. Although principles could be described
as indeterminate, a traditionalist may or may not be
likely to support them.
5. ANSWERS AND EXPLANATIONS
Choice (A) is correct. "Spontaneity" is uncontrolled and
impulsive behavior. "Rigidity" means stiffness. If one
were to insert these terms into the text, the sentence
would read "The author constructed a scenario in which
playful, creative children are rewarded for their
spontaneity and strict, dour adults are punished for their
rigidity." The first missing term should be associated
with creativity or playfulness, and the second should be
related to "strict" and "dour," which mean limiting and
stern. Spontaneity is often associated with creativity,
and someone who is strict or unyielding could aptly be
described as exhibiting rigidity.
Choice (B) is incorrect. "Digressions" are wanderings.
"Mirth" is gladness. If one were to insert these terms
Page 9 of 21
Test 11: Critical Reading Answers
into the text, the sentence would read "The author
constructed a scenario in which playful, creative
children are rewarded for their digressions and strict,
dour adults are punished for their mirth." The first
missing term should be associated with creativity or
playfulness, and the second should be related to "strict"
and "dour," which mean limiting and stern. Mirth does
not work in the sentence because it is not a quality of
strict, dour people.
Choice (C) is incorrect. "Solemnity" means gloomy
seriousness. "Malice" means hatred. If one were to
insert these terms into the text, the sentence would
read "The author constructed a scenario in which
playful, creative children are rewarded for their
solemnity and strict, dour adults are punished for their
malice." The first missing term should be associated
with creativity or playfulness, and the second should be
related to "strict" and "dour," which mean limiting and
stern. Solemnity is the opposite of playfulness, and thus
could not be the answer.
Choice (D) is incorrect. "Inflexibility" means the state of
being rigid or unyielding. "Rigor" means strictness. If
one were to insert these terms into the text, the
sentence would read "The author constructed a
scenario in which playful, creative children are
rewarded for their inflexibility and strict, dour adults are
punished for their rigor." The first missing term should
be associated with creativity or playfulness, and the
second should be related to "strict" and "dour," which
mean limiting and stern. Playful, creative children would
most likely not be inflexible.
Choice (E) is incorrect. "Improvisations" are things
created without planning. "Buoyancy" means the state
of being light or cheerful. If one were to insert these
terms into the text, the sentence would read "The
author constructed a scenario in which playful, creative
children are rewarded for their improvisations and strict,
dour adults are punished for their buoyancy." The first
missing term should be associated with creativity or
playfulness, and the second should be related "strict"
and "dour," which mean limiting and stern. Although
improvisations works in the sentence, the second word
does not. Buoyancy is not associated with strict, dour
people.
Choice (A) is incorrect. "Joyous" means happy.
"Conventional" means lacking originality or individuality.
If one were to insert these terms into the text, the
sentence would read "Although usually warm and
joyous in greeting friends, Lauren was too reserved
ever to be truly conventional." The first missing term is
similar in meaning to "warm," which, in this instance,
means affectionate. The second word should have a
meaning that is the opposite of "reserved," which
means restrained or shy. "Conventional" is not the
opposite of reserved.
Choice (C) is incorrect. "Restrained" means kept under
control. "Gracious" means friendly. If one were to insert
these terms into the text, the sentence would read
"Although usually warm and restrained in greeting
friends, Lauren was too reserved ever to be truly
gracious." The first missing term is similar in meaning to
"warm," which, in this instance, means affectionate. The
second word should have a meaning that is the
opposite of "reserved," which means restrained or shy.
The word "restrained" does not suggest affection.
Choice (D) is incorrect. "Dismissive" describes an
attitude of offhand rejection. "Ebullient" means full of
high spirits. If one were to insert these terms into the
text, the sentence would read "Although usually warm
and dismissive in greeting friends, Lauren was too
reserved ever to be truly ebullient." The first missing
term is similar in meaning to "warm," which, in this
instance, means affectionate. The second word should
have a meaning that is the opposite of "reserved,"
which means restrained or shy. Lauren would not be
both warm and dismissive when she greets friends,
since the these are two opposite attitudes.
Choice (E) is incorrect. "Genial" means cheerful.
"Antisocial" means unsociable. If one were to insert
these terms into the text, the sentence would read
"Although usually warm and cordial in greeting friends,
Lauren was too reserved ever to be truly effusive." The
first missing term is similar in meaning to "warm,"
which, in this instance, means affectionate. The second
word should have a meaning that is the opposite of
"reserved," which means restrained or shy. One would
not say that a person is too reserved to be antisocial.
An extremely reserved person might be considered
antisocial.
6. ANSWERS AND EXPLANATIONS
7. ANSWERS AND EXPLANATIONS
Choice (B) is correct. "Cordial" means warm and
friendly. "Effusive" means unrestrained in emotional
expression. If one were to insert these terms into the
text, the sentence would read "Although usually warm
and cordial in greeting friends, Lauren was too reserved
ever to be truly effusive." The first missing term is
similar in meaning to "warm," which, in this instance,
means affectionate. The second term should have a
meaning that is the opposite of "reserved," which
means restrained or shy. "Cordial" means socially warm
and "effusive" is the opposite of reserved.
Choice (A) is correct. "Malleable" means capable of
being changed or controlled. If one were to insert this
term into the text, the sentence would read "Legal
scholars argue that when 'justice' is interpreted too
broadly, the concept becomes malleable, easily
changed and controlled by outside forces." "Malleable"
properly describes a "justice" that can be altered by
outside forces.
Choice (B) is incorrect. "Influential" means powerful or
having an effect on others. If one were to insert this
Page 10 of 21
Test 11: Critical Reading Answers
term into the text, the sentence would read "Legal
scholars argue that when 'justice' is interpreted too
broadly, the concept becomes influential, easily
changed and controlled by outside forces." An
"influential" concept is one that changes outside forces,
not one that is changed by outside forces.
Choice (C) is incorrect. "Coherent" means well ordered
or clear. If one were to insert this term into the text, the
sentence would read "Legal scholars argue that when
'justice' is interpreted too broadly, the concept becomes
coherent, easily changed and controlled by outside
forces." It makes no sense to imply that a concept can
be easily changed or controlled because it is clear.
Choice (D) is incorrect. "Felicitous" means well suited
or particularly appropriate to a situation. If one were to
insert this term into the text, the sentence would read
"Legal scholars argue that when 'justice' is interpreted
too broadly, the concept becomes felicitous, easily
changed and controlled by outside forces." If something
is well suited, it makes no sense to describe it as easily
changed or controlled.
Choice (E) is incorrect. "Prosaic" means ordinary. If one
were to insert this term into the text, the sentence
would read "Legal scholars argue that when 'justice' is
interpreted too broadly, the concept becomes prosaic,
easily changed and controlled by outside forces." An
ordinary concept might be a concept that is easily
changed or controlled, but it is not necessarily so.
Choice (C) is incorrect. "Melodious" means sweetsounding or full or melody. If one were to insert this
term into the text, the sentence would read "The
instructor's voice was so melodious that most students
preferred taking a test over listening to its grating
sound." The missing term describes a voice that has a
"grating sound." A melodious voice is the opposite of a
grating one.
Choice (D) is incorrect. "Muted" means muffled or
softened. If one were to insert this term into the text, the
sentence would read "The instructor's voice was so
muted that most students preferred taking a test over
listening to its grating sound." The missing term
describes a voice that has a "grating sound." A muted
voice is the opposite of a grating voice.
9. ANSWERS AND EXPLANATIONS
Choice (D) is correct. The passage does not appeal to
reason but to emotion and to the senses. It describes
the feelings and impressions associated with the
physical appearance of the hotel.
Choice (A) is incorrect. The passage does use visual
imagery, such as the descriptions of the padlocked
doors and the reflections in their glass panels.
Choice (B) is incorrect. The sounds of the shutter hinge
and the wind are auditory descriptions, or descriptions
of sound, that are compared to music.
8. ANSWERS AND EXPLANATIONS
Choice (E) is correct. "Strident" means loud, harsh and
discordant. If one were to insert this term into the text,
the sentence would read "The instructor's voice was so
strident that most students preferred taking a test over
listening to its grating sound." The missing term
describes a voice that has a "grating sound." A strident
sound is synonymous with a grating sound.
Choice (A) is incorrect. "Receptive" means open and
responsive to ideas or suggestions. If one were to
insert this term into the text, the sentence would read
"The instructor's voice was so receptive that most
students preferred taking a test over listening to its
grating sound." The missing term describes a voice that
has a "grating sound." A grating sound has nothing to
do with being receptive. Furthermore, no voice could be
described as receptive.
Choice (B) is incorrect. "Cajoling" means persuading or
coaxing. If one were to insert this term into the text, the
sentence would read "The instructor's voice was so
cajoling that most students preferred taking a test over
listening to its grating sound." The missing term
describes a voice that has a "grating sound." A cajoling
sound is not synonymous with a grating sound. A
person who is cajoling would most likely use a soft
voice, not a grating one.
Choice (C) is incorrect. The lonely, vast land outside the
hotel is contrasted with the happy crowd of "best
friends" that the hotel "seems to promise you."
Choice (E) is incorrect. Through hypothetical musings,
or imaginative reflections, the author describes the
empty, padlocked hotel as if it were the scene of a
party.
10. ANSWERS AND EXPLANATIONS
Choice (C) is correct. These lines convey a feeling of
melancholy, or sadness, because of the "hurt" that
haunts the imaginary cheerfulness of the inside of the
hotel.
Choice (A) is incorrect. Earlier in the passage, the
hotel's inside was imagined as lighthearted and happy,
but this is contradicted by the "hurt jamming," or pain
pressing, inside the "halls" and "closed-up rooms" of
the hotel.
Choice (B) is incorrect. Bewilderment, or confusion, is
not conveyed in the description of the hotel's sadness.
Rather, there is a degree of clarity expressed in the
recognition of the sadness that fills the hotel.
Choice (D) is incorrect. Even though nostalgia, or a
longing for the past, is present elsewhere in the
Page 11 of 21
Test 11: Critical Reading Answers
passage, there is no longing for the hotel's "hurt" and
sadness expressed in lines 10-12.
with ideas that have not been either proven or
disproven.
Choice (E) is incorrect. There is no feeling of
detachment, or lack of involvement, in the emotionally
charged description of the "hurt" the author imagines
"jamming those halls and closed-up rooms."
Choice (D) is incorrect. Diffusionists may embrace
theories that claim America was discovered by
accident, but this is not mentioned in the passage.
Choice (E) is incorrect. The passage does not describe
diffusionists as being concerned with rivalries between
new technologies and old ones.
11. ANSWERS AND EXPLANATIONS
Choice (A) is correct. "Comprehensiveness" is the
quality of being all-inclusive and wide-ranging, covering
many ideas—just as an umbrella might cover, or
stretch, over several objects. In this context, "umbrella"
refers to "diffusion theory," which covers or includes
within itself "various alternative theories" (line 2) about
the discovery of America.
Choice (B) is incorrect. "Impenetrable" refers to
something that cannot be penetrated. It makes sense
that an umbrella would be described as "impenetrable"
by rain, but the theories mentioned in the passage have
been penetrated, or shown to be flawed, be
researchers: "if you scrutinize any specific claim, it
melts away" (lines 11-12). Thus they cannot be
described as "impenetrable."
Choice (C) is incorrect. The utility, or usefulness, of an
umbrella is not the quality featured in the passage. The
passage uses the term "umbrella" to refer to the fact
that the "diffusion theory" comprises "various alternative
theories" (line 2).
Choice (D) is incorrect. Even though an umbrella is an
ordinary object, the passage does not refer to the
ordinariness of the "diffusion theory," but to its
comprehensiveness.
Choice (E) is incorrect. The passage explains that
"diffusion theory" includes many alternative theories.
The notion of foresight is not discussed in the passage.
13. ANSWERS AND EXPLANATIONS
Choice (A) is correct. The passages differ substantially
on the question of whether gender determines how
individuals communicate with each other. In Passage 1,
the author states, "I believe that there are gender
differences in ways of speaking, and we need to
identify and understand them" (lines 10-12). The author
of Passage 2 argues that "this approach has a number
of limitations" (line 75), and stresses that individuals
need to move toward "an understanding of gender as
something people do in social interaction" (lines 64-66).
Passage 2 sees gender as a fluid component of social
interactions that anyone can adopt depending on the
situation, whereas Passage 1 sees gender as a fixed
determinant of communication styles.
Choice (B) is incorrect. Both passages give examples
of social behaviors in childhood, and both assume that
these behaviors are at least partly learned.
Choice (C) is incorrect. Passage 1 implies that a study
of conversational styles between adults is possible and
desirable, but Passage 2 does not address this topic.
Choice (D) is incorrect. Passage 1 discusses the role of
gender in shaping how children approach activities
such as conversation; Passage 2 does not deny that
gender is a factor in children's lives, but suggests that
the situation is more complex than sociolinguists claim.
The passages do not primarily address children's
playtime activities.
12. ANSWERS AND EXPLANATIONS
Choice (C) is correct. This assertion uses the great
quantity of UFO sightings as evidence that at least one
of them must be authentic. The quantity of the
sightings, or the number of times people claim to see
UFOs, is emphasized over the quality of the sightings
(a high-quality sighting is one that could be verified by
independent experts). This is the same strategy that
diffusionists use; diffusionists also "emphasize quantity
over quality" (line 13).
Choice (A) is incorrect. Although diffusionists cannot at
this point say which explorer beat Columbus,
diffusionists do not conclude that proof is impossible or
irrelevant.
Choice (B) is incorrect. The Wright Brothers decisively
disproved a theory, whereas diffusion theory has to do
Choice (E) is incorrect. The authors both agree that
society is very interested in gender roles.
14. ANSWERS AND EXPLANATIONS
Choice (D) is correct. Passage 1's argument is that
gender-specific communication differences account for
misunderstandings between men and women. The
author asserts the value of the sociolinguistic approach
to the issue of gender differences, arguing that it is an
approach that should improve understanding between
men and women.
Choice (A) is incorrect. Passage 1 does not present the
history of the controversy over gender differences, but
rather explains how a sociolinguistic approach to the
problem will help people deal with these differences.
Page 12 of 21
Test 11: Critical Reading Answers
Choice (B) is incorrect. Passage 1 does acknowledge
that some scholars are "reluctant," or unwilling, to show
ways in which men and women are different, but the
passage is not primarily concerned with the errors of
others.
Choice (A) is incorrect. Recognizing gender differences
would not cause people to exaggerate their similarities.
It would make men and women aware of their
differences, which would lead to improved
understanding between people.
Choice (C) is incorrect. Passage 1 does not suggest
changes in children's activities, but rather describes
how the different cultures of boys and girls lead them to
communicate differently.
Choice (B) is incorrect. According to Passage 1,
recognizing gender differences would decrease
dissatisfaction, not increase it.
Choice (E) is incorrect. Passage 1 does not discuss
any recent discovery.
15. ANSWERS AND EXPLANATIONS
Choice (B) is correct. Passage 1 explains that some
scholars have been "reluctant to show ways in which
[men and women] are different, because differences
between two groups have so often been used to 'justify'
unequal treatment" (lines 2-5). The author of Passage 1
appreciates the good intentions of those scholars, but
ultimately disagrees with them. The author clearly
believes that acknowledging gender differences will
help people relate to members of the opposite sex.
Choice (A) is incorrect. Far from predicting that the
scholars' work will eventually be accepted, Passage 1
discounts the belief that there are no differences
between women and men by saying, "it simply isn't
so" (line 10).
Choice (C) is incorrect. Passage 1 does not suggest
that the scholars are objective, or unbiased, about
gender differences; rather, it implies that their "desire to
affirm that women and men are completely equal" (lines
1-2) has affected their observations of people's actual
behavior.
Choice (D) is incorrect. The author of Passage 1 firmly
disagrees with the theory that there is no difference
between women and men.
Choice (E) is incorrect. Passage 1 suggests the
opposite: that the scholars' work attempts to cure social
injustice by denying differences between men and
women. There is no sense that the scholars' work might
be used to excuse injustice.
16. ANSWERS AND EXPLANATIONS
Choice (D) is correct. Passage 1's approach to gender
difference is concerned with men's and women's close
relationships with spouses, siblings and friends.
According to the passage, "recognizing gender
differences in conversational styles would free
individuals from the burden of an inappropriate sense of
being at fault" (lines 16-18). In other words, this
recognition would keep people from blaming
themselves (or others) for gender-based
misunderstandings.
Choice (C) is incorrect. While the passage does not
seek to justify unequal treatment and opportunity for
men and women, neither is it concerned with promoting
equal treatment. Rather, it argues for understanding
gender-specific communication patterns in order to
improve relationships.
Choice (E) is incorrect. The passage is not concerned
with how research on gender should be conducted, but
with improving the way men and women relate to each
other in conversation.
. ANSWERS AND EXPLANATIONS
Choice (A) is correct. Passage 1 describes how girls
seek to foster intimacy, or closeness, with others
through communication. Little girls are said to
emphasize "displaying similarities and matching
experiences" (lines 38-39) in their conversations. The
passage points out that for girls, the purpose of
language is "establishing connection and negotiating
relationships" (lines 40-41).
Choice (B) is incorrect. The passage does not mention
the idea that girls try to establish conversational rules;
instead, it argues that they use communication as a
way of establishing relationships through sharing
interests and ideas.
Choice (C) is incorrect. Lines 36-41 state that instead of
seeking to impart information, girls focus on "displaying
similarities and matching experiences" (lines 38-39) in
their conversations.
Choice (D) is incorrect. Nostalgic feelings, or feelings of
longing for the past, are not mentioned in the
description of girls' communication patterns.
Choice (E) is incorrect. According to the passage, girls
are interested in establishing rapport, or creating
agreement. The passage does not describe either girls
or boys as seeking to be objective, or unbiased, in their
conversations.
18. ANSWERS AND EXPLANATIONS
Choice (A) is correct. The claim made in lines 27-31 of
Passage 1 is that "boys and girls grow up in what are
essentially different cultures," so any challenge must
address that claim directly. The author of Passage 2
states that children have "countless experiences
communicating with people of both sexes" (line 86).
The author of Passage 2 would thus argue that children
Page 13 of 21
Test 11: Critical Reading Answers
do not grow up in a single-gender culture.
Choice (B) is incorrect. The author of Passage 2 does
not discount sociolinguistic research by suggesting that
children deceive adults, but by suggesting that
researchers deceive themselves.
Choice (C) is incorrect. Passage 2 argues that gender
differences have not been studied scientifically, or
objectively, by researchers. However, the author of
Passage 2 does not claim that this task is impossible.
Choice (D) is incorrect. The claim made in lines 27-31
of Passage 1 is that "boys and girls grow up in what are
essentially different cultures." The author of Passage 2
would not be likely to address this claim by arguing that
men and women experience less conflict than some
scholars assume.
Choice (E) is incorrect. There is no mention in either
passage of how the behavior of children has changed
in recent years. Neither passage is concerned with
changes over time.
19. ANSWERS AND EXPLANATIONS
Choice (B) is correct. According to the author,
stereotypes "may dictate what we notice and bias our
perceptions in the direction of expectation" (lines
47-48). In other words, when we meet people, gender
stereotypes may cause us to look for the qualities we
expect those people to have, and to focus on those
qualities instead of others. These stereotypes make it
difficult to observe people with a fair, unbiased eye;
they distort, or skew, our view of people.
recognizing gender differences in conversational styles
would free individuals from the burden of an
inappropriate sense of being at fault" (lines 16-18).
Choice (A) is incorrect. The "scholars" in line 2 are
reluctant, or unwilling, to show ways in which men and
women are different. The research work described in
lines 48-51 assumes that there are gender differences
and asks that they be acknowledged.
Choice (B) is incorrect. The dissatisfied women and
men mentioned in line 19 most likely do not assume
that gender differences account for their
misunderstandings. The "researchers" in Passage 2
are trying to help such people.
Choice (C) is incorrect. The "noted scholar" mentioned
in line 66 does not assume that gender differences
determine the way a person acts. This scholar's idea is
very different from the claim of the researchers
mentioned in lines 48-51: that acknowledging gender
differences will help men and women communicate.
Choice (E) is incorrect. The author of Passage 2 does
not assume that acknowledging gender differences will
help men and women communicate. This author is
concerned about the harm done by gender stereotypes.
21. ANSWERS AND EXPLANATIONS
Choice (A) is incorrect. The passage does not mention
the feelings of people being stereotyped.
Choice (C) is correct. The sentence preceding the
quote argues that we need to think of gender "as
something people do in social interaction" (lines 65-66).
The quotation, by pointing out that "in particular
contexts people do feminine, in others, they do
masculine," elaborates on the idea that people's
behavior is determined by situation rather than by
gender.
Choice (C) is incorrect. Passage 2 does not mention
the use of stereotypes to justify unequal treatment of
men and women.
Choice (A) is incorrect. The quotation is not about
someone's personal experience; it is a statement about
people's experiences in general.
Choice (D) is incorrect. The author of Passage 2 may
believe that miscommunication between men and
women can be caused by stereotyping; however, lines
47-48 refer to the ways gender stereotypes distort
people's views of one another.
Choice (B) is incorrect. The quotation does not provide
any kind of example; it is a generalization, or broad
idea.
Choice (E) is incorrect. Passage 2 states that gender
stereotypes influence our perceptions about others. It
does not discuss what these stereotypes reveal about
those who believe in them.
Choice (D) is incorrect. Instead of signaling a new topic,
this quotation extends the topic: the idea that gender is
"something people do" that varies with the situation.
Choice (E) is incorrect. Even though this is a quotation
by a "noted scholar," it does not mention any recent
research data.
20. ANSWERS AND EXPLANATIONS
22. ANSWERS AND EXPLANATIONS
Choice (D) is correct. The assumptions of the
researchers mentioned in lines 48-51 are very similar to
those of the author of Passage 1. Both want "to
elucidate gender differences in order to help women
and men understand and respond to each other better."
As the author of Passage 1 states, "it is clear to me that
Choice (E) is correct. According to Passage 2, in the
attempt to highlight the differences between boys and
girls, some scholars are "singling out those children
who fit common gender stereotypes and marginalizing
others" (lines 78-80). By studying children who tend to
Page 14 of 21
Test 11: Critical Reading Answers
behave according to stereotypes, or "narrow
preconceptions of behavior," those scholars can
construct a picture that highlights the differences in the
cultures of boys and girls.
Choice (A) is incorrect. Passage 2 does not say
anything about children who are eager to interact with
strangers.
Choice (B) is incorrect. Passage 2 does not suggest
that scholars study children who are in strong need of
approval from adults.
Choice (C) is incorrect. The scholars discussed in
Passage 2 study children's behavior and attitudes
toward each other, not toward adults or figures of
authority. Furthermore, there is no mention of rebellion
or creative behavior in Passage 2.
Choice (D) is incorrect. Passage 2 does not suggest
that children in the studies have highly idiosyncratic
personalities, or that they have very particular individual
characteristics. On the contrary, the author states that
the studies focus on children who fit common gender
stereotypes while ignoring children who do not act in
expected ways.
23. ANSWERS AND EXPLANATIONS
Choice (C) is correct. The author cites these children as
examples of children who do not "conform to," or fit in
with, traditional gender stereotypes. The "boys who
excel at caring for younger siblings" are demonstrating
a quality more associated with girls, the author implies.
Likewise, the child who displays dominance with a
younger playmate and shows deference to an older
friend is behaving according to the situation, not
according to gender.
Choice (A) is incorrect. Caring for younger siblings and
displaying dominance or deference might be adult
behaviors that these children are imitating.
Choice (B) is incorrect. The passage doesn't say
whether these children get along with their peers. The
author's main point is that they do not act according to
gender stereotypes.
are thus constant and predictable. The author states: "I
believe that there are gender differences in ways of
speaking, and we need to identify and understand
them" (lines 10-12). Passage 2, on the other hand,
states that gender communication styles are fluid and
change according to the individual and the situation:
"people display contradictory behaviors as they
encounter different social norms and pressures" (lines
69-70).
Choice (A) is incorrect. In Passage 1, the author
mentions that boys tend to make competitive
statements of achievement. However, Passage 1 does
not argue that all styles are based on competition, and
Passage 2 does not suggest that all conversational
styles are a form of cooperation.
Choice (B) is incorrect. Passage 1 does not argue that
styles are a burden, but rather that understanding them
can help improve understanding between men and
women. Passage 2 argues against the existence of
gender specific styles altogether; it points out that
thinking in terms of "differences rather than similarities"
is not helpful.
Choice (C) is incorrect. Passage 1 does claim that
styles are semantic, in that they have to do with
meaning in language. However, Passage 2 argues that
people act and speak according to what they have
learned to do in different situations. This has nothing to
with conversational styles being whimsical.
Choice (E) is incorrect. Passage 1 does not state that
conversational styles are random, or a result of chance,
but that they are determined by gender. Furthermore,
Passage 2 argues that closer scrutiny will show that
styles of behavior in general are less dependent on
gender than is commonly supposed.
----------------------------------------------------------
Section 8
1. ANSWERS AND EXPLANATIONS
24. ANSWERS AND EXPLANATIONS
Choice (A) is correct. "Admired" means thought highly
of. "Embraced" means held closely. If one were to
insert these terms into the text, the sentence would
read "Originally admired mainly by young, urban
audiences, rap music was ultimately embraced by its
appreciative listeners of all ages across the country."
The phrase "appreciative listeners of all ages" indicates
that rap music's fan base expanded from what it was
"originally." The correct response is the only one in
which both missing terms have a positive meaning,
indicating that the appreciation began with one group
and expanded to include a much larger group. Both
"admired" and "embraced" are positive terms.
Choice (D) is correct. According to Passage 1,
communication styles are determined by gender and
Choice (B) is incorrect. "Performed" means acted out or
achieved. "Condemned" means disapproved of. If one
Choice (D) is incorrect. There is nothing to suggest that
these children "mock," or make fun of, adults'
expectations of them, or that they are paying attention
to adults at all.
Choice (E) is incorrect. The passage suggests that
these children, like most children, actually "have
countless experiences communicating with people of
both sexes" (lines 86-87).
Page 15 of 21
Test 11: Critical Reading Answers
were to insert these terms into the text, the sentence
would read "Originally performed mainly by young,
urban audiences, rap music was ultimately condemned
by its appreciative listeners of all ages across the
country." The phrase "appreciative listeners of all ages"
indicates that rap music's fan base expanded from what
it was "originally." The correct response is the only one
in which both missing terms have a positive meaning,
indicating that the appreciation began with one group
and expanded to include a much larger group.
Audiences do not typically perform music, and listeners
would not be described as appreciative of something
they are condemning.
Choice (C) is incorrect. "Derided" means ridiculed.
"Ignored" means disregarded. If one were to insert
these terms into the text, the sentence would read
"Originally derided mainly by young, urban audiences,
rap music was ultimately ignored by its appreciative
listeners of all ages across the country." The phrase
"appreciative listeners of all ages" indicates that rap
music's fan base expanded from what it was
"originally." The correct response is the only one in
which both missing terms have a positive meaning,
indicating that the appreciation began with one group
and expanded to include a much larger group. Both
"derided" and "ignored" have a negative, not a positive,
connotation. Music would not be both appreciated and
ignored by the same group of listeners.
Choice (D) is incorrect. "Appropriated" means
something is taken and used. "Relinquished" means
abandoned. If one were to insert these terms into the
text, the sentence would read "Originally appropriated
mainly by young, urban audiences, rap music was
ultimately relinquished by its appreciative listeners of all
ages across the country." The phrase "appreciative
listeners of all ages" indicates that rap music's fan base
expanded from what it was "originally." The correct
response is the only one in which both missing terms
have a positive meaning, indicating that the
appreciation began with one group and expanded to
include a much larger group. Listeners would not
relinquish the music they appreciate.
Choice (E) is incorrect. "Applauded" means approved
of. "Instigated" means provoked. If one were to insert
these terms into the text, the sentence would read
"Originally applauded mainly by young, urban
audiences, rap music was ultimately instigated by its
appreciative listeners of all ages across the country."
The phrase "appreciative listeners of all ages" indicates
that rap music's fan base expanded from what it was
"originally." The correct response is the only one in
which both missing terms have a positive meaning,
indicating that the appreciation began with one group
and expanded to include a much larger group.
Listeners would not have instigated rap music.
2. ANSWERS AND EXPLANATIONS
Choice (E) is correct. "Modesty" means humility. "A
dilettante" is someone who merely dabbles in a subject.
If one were to insert these terms into the text, the
sentence would read "It was out of modesty that
Professor Green, the author of several highly respected
books in his field, described himself to his colleagues
as a dilettante." The correct response for the second
missing term will be the result of Professor Green's
feelings as described in the first missing term. Because
Professor Green is highly respected in his field, his
description of himself as a dilettante is simply modest;
he describes himself at less than his true value.
Choice (A) is incorrect. "Embarrassment" means selfconsciousness. "A paragon" is a model of excellence. If
one were to insert these terms into the text, the
sentence would read "It was out of embarrassment that
Professor Green, the author of several highly respected
books in his field, described himself to his colleagues
as a paragon." The correct response for the second
missing term will be the result of Professor Green's
feelings as described in the first missing term.
Professor Green would not describe himself as a
paragon, proclaiming his excellence, as a result of his
embarrassment.
Choice (B) is incorrect. "Magnanimity" means forgiving.
"An avenger" is one who seeks revenge. If one were to
insert these terms into the text, the sentence would
read "It was out of magnanimity that Professor Green,
the author of several highly respected books in his field,
described himself to his colleagues as an avenger." The
correct response for the second missing term will be
the result of Professor Green's feelings as described in
the first missing term. Professor Green cannot be both
an avenger and magnanimous.
Choice (C) is incorrect. "Insolence" means disrespect.
"A pedant" is someone overly concerned with the
formal rules of his field of knowledge. If one were to
insert these terms into the text, the sentence would
read "It was out of insolence that Professor Green, the
author of several highly respected books in his field,
described himself to his colleagues as a pedant." The
correct response for the second missing term will be
the result of Professor Green's feelings as described in
the first missing term. Professor Green would be
unlikely to describe himself as a pedant, or someone
overly concerned with the formal rules of his field of
knowledge. If he described himself as a pedant,
however, it would not be as a result of insolence.
Choice (D) is incorrect. "Egotism" means conceit. "An
apprentice" is a beginning learner. If one were to insert
these terms into the text, the sentence would read "It
was out of egotism that Professor Green, the author of
several highly respected books in his field, described
himself to his colleagues as an apprentice." The correct
response for the second missing term will be the result
of Professor Green's feelings as described in the first
missing term. If Professor Green wrote several highly
respected books and was egotistical, he would hardly
describe himself as an apprentice, or beginning learner.
Page 16 of 21
Test 11: Critical Reading Answers
3. ANSWERS AND EXPLANATIONS
Choice (E) is correct. "Denounced" means spoke ill of.
If one were to insert this term into the text, the sentence
would read "Historian Carlo Botta often contradicted
himself, as when he first championed and then
denounced the ideals of the French Revolution." The
word "contradicted" indicates that the correct response
will mean the opposite of championing, or supporting,
ideals. Denouncing ideals is the opposite of
championing them. Thus, to do both would be to
contradict oneself.
Choice (A) is incorrect. "Invoked" means to call upon. If
one were to insert this term into the text, the sentence
would read "Historian Carlo Botta often contradicted
himself, as when he first championed and then invoked
the ideals of the French Revolution." The word
"contradicted" indicates that the correct response will
mean the opposite of championing, or supporting,
ideals. For Botta to invoke the Revolution's ideals after
having championed them would not be a contradiction.
Choice (B) is incorrect. "Investigated" means
researched. If one were to insert this term into the text,
the sentence would read "Historian Carlo Botta often
contradicted himself, as when he first championed and
then investigated the ideals of the French Revolution."
The word "contradicted" indicates that the correct
response will mean the opposite of championing, or
supporting, ideals. While it might have been better for
Botta to investigate the ideals before he championed
them, his investigation doesn't mean he is contradicting
himself.
Choice (C) is incorrect. "Conceived" means thought up.
If one were to insert this term into the text, the sentence
would read "Historian Carlo Botta often contradicted
himself, as when he first championed and then
conceived the ideals of the French Revolution." The
word "contradicted" indicates that the correct response
will mean the opposite of championing, or supporting,
ideals. Botta could not have conceived the ideals of the
French Revolution after having championed them. He
would have conceived them and later championed
them.
Choice (D) is incorrect. "Coveted" means strongly
desired. If one were to insert this term into the text, the
sentence would read "Historian Carlo Botta often
contradicted himself, as when he first championed and
then coveted the ideals of the French Revolution." The
word "contradicted" indicates that the correct response
will mean the opposite of championing, or supporting,
ideals. Coveting ideals may be odd; however, coveting
and championing ideals is not a contradiction.
insert this term into the text, the sentence would read
"Luisa worked with extreme precision, a
meticulousness that served her well in her law career."
A person who works with extreme precision would be
properly described as demonstrating "meticulousness."
Choice (B) is incorrect. "Effrontery" means offensive
boldness. If one were to insert this term into the text,
the sentence would read "Luisa worked with extreme
precision, an effrontery that served her well in her law
career." Working with precision is very different from the
type of shameless or offensive behavior that is termed
"effrontery."
Choice (C ) is incorrect. "Inhibition" means something
that holds a person back. If one were to insert this term
into the text, the sentence would read "Luisa worked
with extreme precision, an inhibition that served her
well in her law career." As the sentence makes clear,
working precisely allowed Luisa to succeed; it did not
hold her back.
Choice (D) is incorrect. "Litigiousness" means a
tendency to engage in lawsuits. If one were to insert
this term into the text, the sentence would read "Luisa
worked with extreme precision, a litigiousness that
served her well in her law career." Although "litigious" is
a word that would describe a lawyer, it does not mean
extreme precision.
Choice (E) is incorrect. "Impetuousness" means
impulsiveness. If one were to insert this term into the
text, the sentence would read "Luisa worked with
extreme precision, an impetuousness that served her
well in her law career." Work that is done on impulse is
probably not performed with extreme precision.
5. ANSWERS AND EXPLANATIONS
Choice (D) is correct. "To burgeon" means to grow and
flourish. If one were to insert this term into the text, the
sentence would read "In 1916 Yellowstone National
Park had only 25 bison, but the population has since
burgeoned to more than 2,000." Since the park began
with 25 bison and now has more than 2,000, the correct
answer is the word that most closely means
"increased." "Burgeoned" and "increased" are similar in
meaning.
Choice (A) is incorrect. "To disperse" means to spread
out. If one were to insert this term into the text, the
sentence would read "In 1916 Yellowstone National
Park had only 25 bison, but the population has since
dispersed to more than 2,000." Since the park began
with 25 bison and now has more than 2,000, the correct
answer is the word that most closely means
"increased." However, "dispersed" and "increased" are
not similar in meaning.
4. ANSWERS AND EXPLANATIONS
Choice (A) is correct. "Meticulousness" means
excessive care or attention to detail. If one were to
Choice (B) is incorrect. "To mediate" means to settle a
dispute between conflicting parties. If one were to insert
this term into the text, the sentence would read "In
Page 17 of 21
Test 11: Critical Reading Answers
1916 Yellowstone National Park had only 25 bison, but
the population has since mediated to more than 2,000."
Since the park began with 25 bison and now has more
than 2,000, the correct answer is the word that most
closely means "increased." However, "mediated" and
"increased" are not similar in meaning. Mediation does
not describe the bison's increase in population.
Choice (C) is incorrect. "To attenuate" means to lessen
in amount or force. If one were to insert this term into
the text, the sentence would read "In 1916 Yellowstone
National Park had only 25 bison, but the population has
since attenuated to more than 2,000." Since the park
began with 25 bison and now has more than 2,000, the
correct answer is the word that most closely means
"increased." However, "attenuated" and "increased" are
not similar in meaning. What happened to the bison
population is the opposite of attenuate.
Choice (E) is incorrect. "To reconcile" means to adapt
or make peace. If one were to insert this term into the
text, the sentence would read "In 1916 Yellowstone
National Park had only 25 bison, but the population has
since reconciled to more than 2,000." Since the park
began with 25 bison and now has more than 2,000, the
correct answer is the word that most closely means
"increased." However, "reconciled" and "increased" are
not similar in meaning.
6. ANSWERS AND EXPLANATIONS
Choice (B) is correct. "Lionized" means celebrated. If
one were to insert this term into the text, the sentence
would read "Though surgeon and researcher Charles
Drew never enjoyed celebrity, he truly deserves to be
lionized for his lifeʼs achievements." This sentence
makes sense because it states that Drew deserves to
be celebrated for his many achievements.
Choice (A) is incorrect. "Mollified" means soothed. If
one were to insert this term into the text, the sentence
would read "Though surgeon and researcher Charles
Drew never enjoyed celebrity, he truly deserves to be
mollified for his lifeʼs achievements." There is no
indication in this sentence that Drew is angry or upset,
thus it is unlikely that he would deserve or need to be
soothed.
Choice (C) is incorrect. "Accosted" means approached
aggressively. If one were to insert this term into the text,
the sentence would read "Though surgeon and
researcher Charles Drew never enjoyed celebrity, he
truly deserves to be accosted for his lifeʼs
achievements." This sentence is illogical because it
does not explain why Drew, a man of many
accomplishments, would deserve to be approached
aggressively.
Choice (D) is incorrect. "Galvanized" means spurred
into action. If one were to insert this term into the text,
the sentence would read "Though surgeon and
researcher Charles Drew never enjoyed celebrity, he
truly deserves to be galvanized for his lifeʼs
achievements." It is unclear why a man of many
achievements would deserve or need to be spurred into
action.
Choice (E) is incorrect. "Vilified" means to be criticized
harshly. If one were to insert this term into the text, the
sentence would read "Though surgeon and researcher
Charles Drew never enjoyed celebrity, he truly
deserves to be vilifed for his lifeʼs achievements." If
Drew did anything to deserve harsh criticism, this
sentence does not mention it.
7. ANSWERS AND EXPLANATIONS
Choice (B) is correct. "Eager" means enthusiastic or
impatient. The author describes her family as "an entire
clan of storytellers competing for a turn on the family
stage" (lines 11-12). This sentence indicates that the
entire family was "eager" for a chance to tell their
stories on the "family stage."
Choice (A) is incorrect. The sentence clearly states that
the mother was just one of "an entire clan of
storytellers" (line 11). There is no indication that anyone
was trying to get her attention.
Choice (C) is incorrect. There is no mention of
household duties or the family's feelings about them in
the passage.
Choice (D) is incorrect. The passage does not indicate
that family members were selective about sharing
information, only that they competed eagerly to be
heard.
Choice (E) is incorrect. The passage refers to the family
members' desire to have their stories heard, regardless
of subject. Further, the only educational
accomplishments mentioned are those of the daughter.
8. ANSWERS AND EXPLANATIONS
Choice (D) is correct. The author writes, "Other kids,
White kids, had forgotten boxes during the week.
They'd brought boxes the next day. I asked for the
same dispensation, but was denied" (lines 24-26).
According to the passage, the author's third grade
teacher arbitrarily made allowances for some students
(White students in particular) and not for others.
Choice (A) is incorrect. The third paragraph presents
the teacher as critical of the author for not having
followed directions, not for having overly grand
ambitions.
Choice (B) is incorrect. The paragraph doesn't indicate
how the teacher feels about her students' interest in
science; it only indicates to what degree and for which
students she is willing to bend her rules.
Choice (C) is incorrect. The teacher does not indicate in
Page 18 of 21
Test 11: Critical Reading Answers
the third paragraph or anywhere else in the passage
that she is concerned about the students' decorum, or
manners.
Choice (E) is incorrect. Nowhere in the passage does
the teacher mention her feelings about injustice or
about the school district. On the contrary, the author
suggests that the teacher herself is guilty of injustice.
9. ANSWERS AND EXPLANATIONS
Choice (B) is correct. "Dispensation" means favor or
courtesy. The sentences immediately preceding line 26
indicate that the author wished to receive the same
"dispensation," the same courtesy, given the "White"
children, namely, being allowed to bring a box the next
day.
Choice (A) is incorrect. The author has already
completed the experiment and does not need "an
additional day to complete the experiment."
Choice (C) is incorrect. The passage indicates that the
dispensation the author is requesting is to bring a box
the following day, not permission to ask her father for
help.
Choice (D) is incorrect. The author does not ask to
leave school early to look for a box, but to bring a box
the next day.
used for this purpose.
Choice (E) is incorrect. The experiment calls for
weighing objects in water; this does not require liquid to
be transported from place to place.
11. ANSWERS AND EXPLANATIONS
Choice (E) is correct. "Righteous indignation" suggests
just or correct anger. The passage states, "Well, my
mother marched me and my armload of buoyant
materials right back into the school" (lines 34-35). This
shows the mother's actions as determined and forceful;
they are part of an angry response to what she sees as
an injustice committed against her daughter.
Choice (A) is incorrect. The mother's decisive action in
seeking out the teacher indicates not bewilderment but
anger and determination.
Choice (B) is incorrect. The mother is not disappointed
but angry and determined to rectify the situation.
Further, there is no sign that the mother is weary of
standing up for her daughter.
Choice (C) is incorrect. The mother is certainly
protective of her daughter's interests, but her attitude is
not one of generosity. Rather, she shows her
determination to right a wrong.
Choice (E) is incorrect. The passage indicates that the
author wants a "dispensation" to bring a box the
following day, not to discuss the experiment with others.
Choice (D) is incorrect. The mother is dismayed but
hardly overwhelmed, as her actions in immediately
returning to the school to confront the teacher clearly
indicate.
10. ANSWERS AND EXPLANATIONS
12. ANSWERS AND EXPLANATIONS
Choice (D) is correct. The author describes the
experiment as a "simple buoyancy experiment" which
involves weighing "each object in the air and then in
water, to prove they weighed less in water" (lines
30-32). Since the experiment calls for weighing objects
in water, the bucket is most likely to be used to hold the
water that is needed.
Choice (A) is correct. The mother's questions, "The box
was the only problem? Just the box? Nothing wrong
with the experiment? An excited eight year old had
forgotten a lousy, stinking box" and "for that she was
out of the running?" (lines 36-40), are part of her
doubting query. The mother clearly believes that the
teacher has placed undue importance on the box. Her
questions are intended to highlight the "absurdity of the
teacher's position."
Choice (A) is incorrect. The experiment involves
weighing particular objects in air and in water (lines
30-32). The author does not mention the use of objects
to counterbalance the weight of other objects.
Choice (B) is incorrect. The author does not mention
the need to steady the scale. Further, the piece of wood
or the brick from the list would be more appropriate
objects to use to steady a scale.
Choice (C) is incorrect. The author's statement that "I
came out of school carrying the pieces of an
experiment" (lines 28-29) suggests that the author does
not have a suitable container to carry the materials for
the project. Because one of the main points of the story
is that the author does not have a suitable container for
her materials, it is clear that the bucket is not being
Choice (B) is incorrect. The questions are being put to
the teacher and not to the child. This is clearly shown
by the continuing dialogue which occurs between
mother and teacher in lines 40-45.
Choice (C) is incorrect. The mother's disdain for the
idea that a box alone should keep her daughter's
experiment out of the competition indicates that she is
not concerned—or at least not interested in expressing
concern to the teacher—about her daughter's
forgetfulness.
Choice (D) is incorrect. The questions are being asked
of the teacher. The mother is more interested in
securing the author's place in the competition than in
Page 19 of 21
Test 11: Critical Reading Answers
reassuring the author about the project's reception.
Choice (E) is incorrect. The mother does not need help
understanding her child's defensiveness about the box,
since her questions indicate that she finds the issue
ridiculous.
13. ANSWERS AND EXPLANATIONS
Choice (B) is correct. In line 23, the box as the teacher
sees it is a necessary element for the presentation of
the experiment at the science fair. In line 38, it is, as the
mother sees it, a "lousy, stinking box that you can get
from the supermarket," and its degree of importance
has shifted from necessary to minor.
Choice (A) is incorrect. The box is still a requirement,
as indicated by the fact that the mother and daughter
find a box for the experiment materials before the
experiment proceeds to the fair.
Choice (C) is incorrect. Neither the teacher nor the
author considered the box a "diversion," or something
that is amusing, and it does not later require a
"desperate search." As the mother points out, a box can
be found almost anywhere (line 39).
Choice (D) is incorrect. The box is never a tool, but a
required carrier for the experiment. At no point is the
box considered a source of entertainment for anyone
involved.
Choice (E) is incorrect. The box is never presented as
either a puzzle or as something that provides clarity
and strength. It is a requirement that later becomes, as
the mother indicates, a petty impediment to her
daughter's potential success.
promotion will have the unintended effect of causing the
interviewers not to praise her, sincerely or otherwise,
but to see her family as "forward and pushy" (line 56).
15. ANSWERS AND EXPLANATIONS
Choice (E) is correct because "blinding" is not used to
mean "sudden" in this context. We can infer that the
mother's instinct is always to defend her daughter and
that her sense of purpose can therefore not be
described as "sudden."
Choice (A) is incorrect because the mother's "blinding"
sense of purpose can be described as "unswerving," or
steady.
Choice (B) is incorrect because the mother's "blinding"
sense of purpose was sufficiently effective to overrule,
and thus stun or dazzle, the teacher.
Choice (C) is incorrect because "overpowering" is a
word that can be used to describe the mother's
"blinding" sense of purpose. Indeed, her argument
overpowers the teacher's objections.
Choice (D) is incorrect because the mother's "blinding"
sense of purpose can certainly be described as
"determined."
16. ANSWERS AND EXPLANATIONS
Choice (E) is correct. The phrase "somebody was
home" is part of a sentence in which the author
describes her mother defending her. For the author, her
mother's vigorous defense is a way of conveying that
the author is important to and supported by her family.
This clearly captures the mother's "sense of
responsibility toward her daughter."
14. ANSWERS AND EXPLANATIONS
Choice (C) is correct. In this context, the word "plug"
means promote or boost. The author uses the word
"plug," a word associated with selling a product, to
express her sense that her mother is over-promoting
her.
Choice (A) is incorrect. The author is not embarrassed
because of a pause in conversation; she is
embarrassed because her mother is openly promoting
her.
Choice (B) is incorrect. In this passage, the author does
not indicate what her feelings are about the boarding
school.
Choice (D) is incorrect. The author feels that her
mother's words and actions are in harmony; both are
intended to promote and protect her daughter.
Choice (E) is incorrect. The author uses the word "plug"
to describe her feelings about what she sees as her
mother's shameless promotion. She worries that this
Choice (A) is incorrect. Undoubtedly the mother wants
to create a comfortable life for her family, but the
phrase in line 62 refers specifically to the mother's
support and defense of her daughter.
Choice (B) is incorrect. Although we can assume that
the mother was eager to learn the results of the
interview, the passage describes only part of the
interview. The passage does not discuss the results of
the interview or the feelings of the participants as to
those results. Further, the phrase in line 62 refers to an
earlier event, not to the interview.
Choice (C) is incorrect. The mother's story indicates
that she feels not despair but determination to confront
injustice toward her daughter.
Choice (D) is incorrect. Although the passage indicates
elsewhere that the mother enjoys attention, the phrase
"somebody was home" refers specifically to her
defense of her daughter.
17. ANSWERS AND EXPLANATIONS
Page 20 of 21
Test 11: Critical Reading Answers
theory.
Choice (A) is correct. These lines indicate the various
reactions, ranging from approval to amusement, of the
story's listeners: "[t]he other mothers nodded
approvingly. My father gave me a wide, clever-girl
smile. The officials from the school looked at me
deadpan" (lines 63-65).
Choice (B) is incorrect. The details in lines 63-66 are
straightforward rather than satirical. The author
interprets the school officials' "deadpan" reaction as
amused, not pompous or arrogant.
Choice (C) is incorrect. Although the other mothers nod
in approval, possibly signaling a sense of unity with the
author's mother, lines 63-66 also stress how the author
herself and the school officials reacted to the mother's
story.
Choice (D) is incorrect. Although the author expresses
embarrassment, the point of lines 63-66 is not to show
her unease, but to show the reactions of everyone in
the room to her mother's story.
Choice (E) is incorrect. Lines 63-66 do not clarify a
previous statement; instead, they indicate the listeners'
reactions to the mother's story.
Choice (E) is incorrect. As the author sees it, the
message of the mother's story is intended for the
school officials and not for her daughter.
19. ANSWERS AND EXPLANATIONS
Choice (A) is correct. The main point of the passage is
to show what kind of person the author's mother is.
Choice (B) is incorrect. The boarding school
admissions process is barely mentioned in the
passage.
Choice (C ) is incorrect. Although the author feels
embarrassed by her mother's story, the passage does
not suggest that she feels hostility, repressed or
otherwise, toward her mother. Rather, she feels
admiration and thanks for her mother and her actions.
Choice (D) is incorrect. The mother's story indicates
that she is well aware of racism; however, this is not the
central purpose of the passage. The central purpose of
the passage is to describe the mother's response to life
and to her family.
Choice (E) is incorrect. The author does not indicate or
suggest that she has become skeptical of human
nature.
18. ANSWERS AND EXPLANATIONS
Choice (B) is correct. In the final paragraph, the author
calls her mother's story "a message about her maternal
concerns, and a way to prove that racism was not some
vanquished enemy" (lines 69-71). She adds, "[w]hen I
was in third grade, Mama could do her maternal
duty" (lines 73-74), and asks, "[w]ho at this new school
would stand up for her child in her stead?" (75-76). The
author sums up the purpose of her mother's story as a
challenge to the school officials to protect her daughter
as fiercely as she has done and will, if necessary,
continue to do.
Choice (A) is incorrect. The author indicates that her
mother's story was "an answer (part rebuke and part
condolence) to the school stories that the admissions
people told, where no parents figured at all" (lines
67-69). Forgiveness and understanding do not figure
prominently in the mother's story.
Choice (C) is incorrect. The author sees the story in
terms of her mother's determination to defend her
daughter rather than as her mother's way of conveying
her personal ideas on hard work.
Choice (D) is incorrect. Although the author describes a
theory about a particular social attitude—one that might
imagine that "racism was not some vanquished enemy,
but a real, live person, up in your face, ready, for no
apparent reason, to mess with your kid" (lines 71-73)—
this theory is one part of her mother's story. The author
sees the story as her mother's way of expressing her
reaction to this theory rather than as a defense of the
Page 21 of 21
© Copyright 2025 Paperzz